Re: Dubbio sull' epsilon delta nei limiti (pt2)

Messaggioda urca » 23/11/2019, 14:46

gugo82 ha scritto:Scusa, ma non è lo stesso?


Diamine, non riesco a vederlo!

$ forall \sigma > 0,\ \exists \delta_(c\sigma) > 0:\quad 0<|x - x_0|<\delta(c\sigma) \Rightarrow |f(x) - l| < c\cdot \sigma$

Risolvendo: $|f(x) - l| < c\cdot \sigma $ da questa discende un intorno bucato di $x_0$: $(x_0-\delta_(c\sigma),x_0+\delta_(c\sigma))$ che mi pare più grande di $(x_0-\delta_sigma,x_0+\delta_sigma)$.
urca
New Member
New Member
 
Messaggio: 23 di 80
Iscritto il: 20/05/2018, 10:32

Re: Dubbio sull' epsilon delta nei limiti (pt2)

Messaggioda gugo82 » 23/11/2019, 15:13

E perché ciò dovrebbe darti fastidio?
Sono sempre stato, e mi ritengo ancora un dilettante. Cioè una persona che si diletta, che cerca sempre di provare piacere e di regalare il piacere agli altri, che scopre ogni volta quello che fa come se fosse la prima volta. (Freak Antoni)
Avatar utente
gugo82
Cannot live without
Cannot live without
 
Messaggio: 22878 di 44972
Iscritto il: 12/10/2007, 23:58
Località: Napoli

Re: Dubbio sull' epsilon delta nei limiti (pt2)

Messaggioda urca » 23/11/2019, 15:22

Perché prendendo il caso c>1, è vero che è un sottocaso, ma basta un controesempio per rendere vana la faccenda. Qundi prendiamo questo c>1 e riprendendo quanto dicevo "a questo punto mettiamo esista una $x\in (x_0-\delta_sigma,x_0+\delta_sigma)$ che non verifichi $|f(x) - l| < sigma$ : questa sarebbe una non esistenza del limite. Tuttavia potrebbe, quella stessa x, verificare $|f(x) - l| < c\sigma$ (che è più grande come intorno). E quindi questa seconda versione di limite mi sembra comprendere dei limiti che per la versione "canonica" non sarebbe vero"

Enon capisco dove cado in fallo
urca
New Member
New Member
 
Messaggio: 24 di 80
Iscritto il: 20/05/2018, 10:32

Re: Dubbio sull' epsilon delta nei limiti (pt2)

Messaggioda gugo82 » 23/11/2019, 15:24

Va bene, ma questo è ovvio.

Il punto della definizione, infatti, non è quello che succede per un fissato $epsilon$, ma quello che accade per ogni $epsilon$.
Sono sempre stato, e mi ritengo ancora un dilettante. Cioè una persona che si diletta, che cerca sempre di provare piacere e di regalare il piacere agli altri, che scopre ogni volta quello che fa come se fosse la prima volta. (Freak Antoni)
Avatar utente
gugo82
Cannot live without
Cannot live without
 
Messaggio: 22879 di 44972
Iscritto il: 12/10/2007, 23:58
Località: Napoli

Re: Dubbio sull' epsilon delta nei limiti (pt2)

Messaggioda urca » 23/11/2019, 15:26

gugo82 ha scritto:Trova un controesempio.


Credo di non saperlo fare, ci ho provato tanto ieri, ma non mi viene mai in mente una funzione che renda vero quanto affermo, eppure ad occhio mi pareva che potrebbe succedere. Data la tua "sfida a trovarlo" deduco di no :-D
Però non capisco il perché, a parte bovinamente provarci e non risucirci :lol:
urca
New Member
New Member
 
Messaggio: 25 di 80
Iscritto il: 20/05/2018, 10:32

Re: Dubbio sull' epsilon delta nei limiti (pt2)

Messaggioda urca » 23/11/2019, 15:43

gugo82 ha scritto:Il punto della definizione, infatti, non è quello che succede per un fissato $epsilon$, ma quello che accade per ogni $epsilon$.


Ho visto ora il tuo edit e rispondo alla seconda versione. Diciamo che sì, riesco a vedere che per ogni epsilon, però non riesco a capire perché non ci debba per forza essere coerenza tra l'epsilon che scelgo e l'epsilon che uso nell'ultima disequazione della definizione.

Insomma se dico per ogni $epsilon$ => $|f(x)-l|<epsilon$ è la stessa cosa di dire per ogni $cepsilon$ => $|f(x)-l|<cepsilon$ mi suona benissimo.

Ma scegliere prima un epsilon qualunque, e poi dire che => $|f(x)-l|<cepsilon$ mi stona, per il motivo degli intorni di cui sopra.


Mi sa che dopo due giorni di miei vacui ragionamenti che mi sono fatto mi tocca capitolare e chiederti come sia la via corretta per mostrare quelle due definizioni che riportasti in esercizio. Non per pigrizia ma per ragionarci sopra e capire dove sbaglio, perché nonostante le tue imbeccate sento di esser fuori strada.
urca
New Member
New Member
 
Messaggio: 26 di 80
Iscritto il: 20/05/2018, 10:32

Re: Dubbio sull' epsilon delta nei limiti (pt2)

Messaggioda gugo82 » 23/11/2019, 15:55

Cioè, ti dà fastidio scegliere un numero positivo $epsilon$ e poi considerare un intorno di semiampiezza $c epsilon$?
Sono sempre stato, e mi ritengo ancora un dilettante. Cioè una persona che si diletta, che cerca sempre di provare piacere e di regalare il piacere agli altri, che scopre ogni volta quello che fa come se fosse la prima volta. (Freak Antoni)
Avatar utente
gugo82
Cannot live without
Cannot live without
 
Messaggio: 22880 di 44972
Iscritto il: 12/10/2007, 23:58
Località: Napoli

Re: Dubbio sull' epsilon delta nei limiti (pt2)

Messaggioda urca » 23/11/2019, 16:36

Uhm non proprio: mi dà fastidio scegliere epsilon e sapere che quell'epsilon dovrebbe permettermi di trovare il delta che controlla le ascisse (questo si fa risolvendo: $|f(x)-l|<epsilon$), e come dicevo prima le x che stanno in quell'intorno di raggio $delta$ risolveranno in generale $|f(x)-l|<epsilon$ ma non $|f(x)-l|<c*epsilon$ (dove l'$epsilon$ di $c*espilon$ è quello che ho scelto all'inizio e per cui ho trovato i delta)

Inoltre oltre a questo mi turba, come dicevo, che se risolvo: $|f(x)-l|<c*epsilon$ trovo un $delta_(c\epsilon)$ in generale diverso da $delta_(epsilon)$ eppure mi dici che non hanno differenza. E la cosa mi manda in crisi :-D

Cioè non riesco a vedere i problemi delle considerazioni e a schiodarmici:
Testo nascosto, fai click qui per vederlo
urca ha scritto:Ci provo sfruttando una lettura di una tua risposta fatta qualche giorno fa che per serendipità ho incontrato e cade a fagiuolo con questo dubbio https://www.matematicamente.it/forum/vi ... 0#p8438453

Comunque senza divagare mi verrebbe da sfruttarla così.
Il problema è che mi si aprono due visioni:

1)
a)

$\forall \epsilon > 0,\ \exists \delta > 0:\quad 0<|x - x_0|<\delta \Rightarrow |f(x) - l| <\epsilon$

Questo vuol dire appunto trovare un insieme non vuoto $A_\epsilon$ il quale intersecato con le x del dominio della funzione mi dia: $|f(x) - l| <\epsilon$ Da questo prendo un intorno bucato del tipo $(x_0-\delta_epsilon,x_0+\delta_epsilon)$ il quale garantisce: $AA x in text(Dom)(f),\ 0<|x - x_0| <delta_epsilon => |f(x) - l| < epsilon$.

b)
Per quanto riguarda: $forall \epsilon > 0, \exists \delta_epsilon > 0:\quad 0<|x - x_0|<\delta_epsilon \Rightarrow |f(x) - l| < c\cdot \epsilon$

Dato che parto da $\forall \epsilon > 0$ identica a quella precedente, risolvendo: $|f(x) - l| <\epsilon$ trovo lo stesso intorno bucato e lo stesso delta di prima. E siccome $c\epsilon>epsilon$ (ipotizzando c>1)

$forall \epsilon > 0,\ \exists \delta > 0:\quad 0<|x - x_0|<\delta \Rightarrow |f(x) - l| <\epsilon=>|f(x) - l| < c\cdot \epsilon $

Il contrario non mi sembra in generale vero, perché seppur epsilon sia QUALUNQUE, in realtà ogni intorno che isolo con la disequazione su $epsilon$ non raggiungerà mai quella di $c\epsilon$.

2)
Vi è un secondo modo di vedere il punto b) e non capisco se questo o il precedente siano corretti: (il dubbio è se il delta vada preso in dipendenza di $epsilon$ o di $c\epsilon$)

b) Dicevamo: $ forall \epsilon > 0,\ \exists \delta_(c\epsilon) > 0:\quad 0<|x - x_0|<\delta(c\epsilon) \Rightarrow |f(x) - l| < c\cdot \epsilon$

Ossia se devo risolvere: $|f(x) - l| < c\cdot \epsilon $ da questa discende per i ragionamenti di cui sopra un intorno bucato di $x_0$: $(x_0-\delta_(c\epsilon),x_0+\delta_(c\epsilon))$ [ovviamente più grande di $x_0-\delta_epsilon,x_0+\delta_epsilon$ del punto a) ).
Ossia arrivo ad avere $AA x in text(Dom)(f),\ 0<|x - x_0| <delta_(c\epsilon) => |f(x) - l| < c\epsilon$

Ebbene in questa seconda visione mi sembra che io fisso un $epsilon$, risolvo la disequazione per $c*epsilon$ e ricavo un intorno più grande della disequazione su $epsilon$ (abbastanza ovvio). A questo punto mettiamo esista una $x\in (x_0-\delta_epsilon,x_0+\delta_epsilon)$ che non verifichi $|f(x) - l| < epsilon$ : questa sarebbe una non esistenza del limite. Tuttavia potrebbe, quella stessa x, verificare $|f(x) - l| < c\epsilon$ (che è più grande come intorno). E quindi questa seconda versione di limite mi sembra comprendere dei limiti che per la versione "canonica" non sarebbe vero!
urca
New Member
New Member
 
Messaggio: 27 di 80
Iscritto il: 20/05/2018, 10:32

Re: Dubbio sull' epsilon delta nei limiti (pt2)

Messaggioda gugo82 » 23/11/2019, 16:52

urca ha scritto:Uhm non proprio: mi dà fastidio scegliere epsilon e sapere che quell'epsilon dovrebbe permettermi di trovare il delta che controlla le ascisse (questo si fa risolvendo: $|f(x)-l|<epsilon$), e come dicevo prima le x che stanno in quell'intorno di raggio $delta$ risolveranno in generale $|f(x)-l|<epsilon$ ma non $|f(x)-l|<c*epsilon$

E grazie… Il $delta$ che trovi o ti consente di controllare lo scostamento dal limite o con $epsilon$ oppure con $c epsilon$, ma non sempre con entrambi.
Infatti, se $c>1$, allora $epsilon < c epsilon $ e perciò il $delta$ che trovi in corrispondenza di $epsilon$ va bene anche per $c epsilon$; viceversa, se $0<c<1$, allora $c epsilon < epsilon $ ed il delta che trovi in corrispondenza di $c epsilon$ va bene pure per $epsilon$.

Le cose si capiscono non chiacchierando, ma dimostrando l’equivalenza delle due proposizioni che ho scritto sopra.
gugo82 ha scritto:Esercizio:
Mostrare che le proposizioni:
\[
\begin{split}
&\forall \varepsilon > 0,\ \exists \delta > 0:\quad 0<|x - x_0|<\delta \Rightarrow |f(x) - l| <\varepsilon \\
&\forall \sigma > 0,\ \exists \rho > 0:\quad 0<|x - x_0|<\rho \Rightarrow |f(x) - l| < c\cdot \sigma \;,
\end{split}
\]
in cui $c >0$ è una costante fissata, sono equivalenti.

Evidentemente, per semplicità formale, sto supponendo $f:RR -> RR$; ma il succo della cosa non cambia per $f:X -> RR$ con $X!=emptyset $.
Vediamo:
Testo nascosto, fai click qui per vederlo
$1=>2$) Supponiamo che:
\[
\forall \varepsilon > 0,\ \exists \delta > 0:\quad 0<|x - x_0|<\delta \Rightarrow |f(x) - l| <\varepsilon
\]
e proviamo che:
\[
\forall \sigma > 0,\ \exists \rho > 0:\quad 0<|x - x_0|<\rho \Rightarrow |f(x) - l| < c\cdot \sigma \;.
\]

Scelto $sigma >0$, in corrispondenza del numero $epsilon := c sigma >0$ per ipotesi esiste un $delta = delta_epsilon = delta_(c sigma) >0$ tale che:
\[
0<|x - x_0|<\delta \Rightarrow |f(x) - l| <\varepsilon = c \sigma \;;
\]
quindi la tesi è verificata prendendo $rho=rho_sigma=delta_(c sigma)$. Vista l’arbitrarietà nella scelta di $sigma$, abbiamo la tesi.

$2=>1$) $1=>2$) Supponiamo che:
\[
\forall \sigma > 0,\ \exists \rho > 0:\quad 0<|x - x_0|<\rho \Rightarrow |f(x) - l| < c\cdot \sigma
\]
e proviamo che:
\[
\forall \varepsilon > 0,\ \exists \delta > 0:\quad 0<|x - x_0|<\delta \Rightarrow |f(x) - l| <\varepsilon \;.
\]

Scelto $epsilon >0$, in corrispondenza del numero $sigma := epsilon/c >0$ per ipotesi esiste un $rho = rho_sigma = rho_(epsilon/c) >0$ tale che:
\[
0<|x - x_0|<\rho \Rightarrow |f(x) - l| < c \sigma = \varepsilon \;;
\]
quindi la tesi è verificata prendendo $delta =delta_epsilon =rho_(epsilon/c)$. Vista l’arbitrarietà nella scelta di $epsilon$, abbiamo la tesi. 8-)
Sono sempre stato, e mi ritengo ancora un dilettante. Cioè una persona che si diletta, che cerca sempre di provare piacere e di regalare il piacere agli altri, che scopre ogni volta quello che fa come se fosse la prima volta. (Freak Antoni)
Avatar utente
gugo82
Cannot live without
Cannot live without
 
Messaggio: 22881 di 44972
Iscritto il: 12/10/2007, 23:58
Località: Napoli

Re: Dubbio sull' epsilon delta nei limiti (pt2)

Messaggioda urca » 23/11/2019, 17:08

Scelto $sigma >0$, in corrispondenza del numero $epsilon := c sigma >0$ per ipotesi esiste un $delta = delta_epsilon = delta_(c sigma) >0$[...]


Quindi quando prendo il "per ogni sigma esiste un delta" non tengo fisso il $sigma$ sempre, in quel passaggio quotato è come se dicessi già dall'inizio "per ogni $c*sigma$".

Dalla definizione, invece, pensavo di dover tenere quel sigma fisso :oops:
Ultima modifica di urca il 23/11/2019, 17:13, modificato 1 volta in totale.
urca
New Member
New Member
 
Messaggio: 28 di 80
Iscritto il: 20/05/2018, 10:32

PrecedenteProssimo

Torna a Analisi matematica di base

Chi c’è in linea

Visitano il forum: Nessuno e 1 ospite